NMTC Problem 3b

If x,y,z are each greater than 1, show that

x4(y1)2+y4(z1)2+z4(x1)248\frac { { x }^{ 4 } }{ { (y-1) }^{ 2 } } +\frac { { y }^{ 4 } }{ { (z-1 })^{ 2 } } +\frac { { z }^{ 4 } }{ { (x-1) }^{ 2 } } \ge 48

This a part of my set NMTC 2nd Level (Junior) held in 2014.

#Algebra #Inequalities #NMTC

Note by Siddharth G
6 years, 7 months ago

No vote yet
1 vote

  Easy Math Editor

This discussion board is a place to discuss our Daily Challenges and the math and science related to those challenges. Explanations are more than just a solution — they should explain the steps and thinking strategies that you used to obtain the solution. Comments should further the discussion of math and science.

When posting on Brilliant:

  • Use the emojis to react to an explanation, whether you're congratulating a job well done , or just really confused .
  • Ask specific questions about the challenge or the steps in somebody's explanation. Well-posed questions can add a lot to the discussion, but posting "I don't understand!" doesn't help anyone.
  • Try to contribute something new to the discussion, whether it is an extension, generalization or other idea related to the challenge.
  • Stay on topic — we're all here to learn more about math and science, not to hear about your favorite get-rich-quick scheme or current world events.

MarkdownAppears as
*italics* or _italics_ italics
**bold** or __bold__ bold

- bulleted
- list

  • bulleted
  • list

1. numbered
2. list

  1. numbered
  2. list
Note: you must add a full line of space before and after lists for them to show up correctly
paragraph 1

paragraph 2

paragraph 1

paragraph 2

[example link](https://brilliant.org)example link
> This is a quote
This is a quote
    # I indented these lines
    # 4 spaces, and now they show
    # up as a code block.

    print "hello world"
# I indented these lines
# 4 spaces, and now they show
# up as a code block.

print "hello world"
MathAppears as
Remember to wrap math in \( ... \) or \[ ... \] to ensure proper formatting.
2 \times 3 2×3 2 \times 3
2^{34} 234 2^{34}
a_{i-1} ai1 a_{i-1}
\frac{2}{3} 23 \frac{2}{3}
\sqrt{2} 2 \sqrt{2}
\sum_{i=1}^3 i=13 \sum_{i=1}^3
\sin \theta sinθ \sin \theta
\boxed{123} 123 \boxed{123}

Comments

Firstly, we have, (a2)20    a24a+40    a24(a1)    a2a14    a4(a1)216 (a-2)^2 \geq 0 \implies a^2 -4a + 4 \geq 0 \implies a^2 \geq 4(a-1) \implies \frac{a^2}{a-1} \geq 4 \implies \frac{a^4}{(a-1)^2} \geq 16

Now, By AM-GM,

x4(y1)2+y4(z1)2+z4(x1)23x4(y1)2y4(z1)2z4(x1)23=3x4(x1)2y4y1)2z4(z1)2331616163=316=48. \frac{x^4}{(y-1)^2} + \frac{y^4}{(z-1)^2} + \frac{z^4}{(x-1)^2} \geq 3\sqrt[3]{\frac{x^4}{(y-1)^2}* \frac{y^4}{(z-1)^2}* \frac{z^4}{(x-1)^2} } = 3\sqrt[3]{\frac{x^4}{(x-1)^2}* \frac{y^4}{y-1)^2}* \frac{z^4}{(z-1)^2} } \geq 3\sqrt[3]{16*16*16} = 3 * 16 = 48.

Siddhartha Srivastava - 6 years, 7 months ago

Log in to reply

Amazing answer! Thank you!

Siddharth G - 6 years, 7 months ago

Log in to reply

Is the AM-GM step necessary? You could just say x4(x1)416\frac{x^4}{(x-1)^4} \geq 16 and so on for y and z and add the 3 inequalities together right?

Josh Banister - 6 years, 5 months ago

Log in to reply

@Josh Banister But in the question, the denominator and the numerator are of different variables, which makes the AM-GM necessary to bring the denominator and the numerator with the same variables together. Thus x4(x1)216\frac { { x }^{ 4 } }{ { (x-1) }^{ 2 } } \ge 16 can only be used after the AM_GM step.

Siddharth G - 6 years, 5 months ago

Log in to reply

@Siddharth G You're right. Just skipped over that for some reason ^o^

Josh Banister - 6 years, 5 months ago
×

Problem Loading...

Note Loading...

Set Loading...